PT1.S3.Q10
To follow directions ... of scientific information") A, B, D, E are necessary assumptions; or ... democracies or not democracies. PT1.S3.Q14
"monetary systems have ...
So for PT1S3 Q6 the prompt says that ... ) Wrong structure, wrong answer D : (A->/B, /A-> ... see). This eliminates C and D because they both have a ...
I was down to A and C, and the answer is A, but I chose C.
I'm having a difficult time understanding why A is the answer and C is wrong.
I mean I thought both could be answers.
http://7sage.com/lsat_explanations/lsat-51-section-3-question-18/
Ok. The explanation JY wrote is awesome, but I still don't understand why B is correct.
I don't understand why A is the correct answer. I chose C. Could someone please explain why the right answer is right and the wrong answers are wrong? Thank you
This question just doesn't seem to click for me any which way I look at it... I don't see how answer choice (A) is the correct answer. If someone has any insight as to how to better approach this question, be my guest! Thanks!
http://7sage.com/ ...
Why is the answer E? I chose A because the 1st paragraph stated, "The problem can be cured by crop rotation, denying the pathogens a suitable host for a period of time."